Integration in polar coordinates | MIT 18.02SC Multivariable Calculus, Fall 2010

Поделиться
HTML-код
  • Опубликовано: 5 окт 2024

Комментарии • 262

  • @rach3834
    @rach3834 4 года назад +27

    This was honestly so so so helpful. From the bottom of my heart, thank you thank you THANK YOU so much. We need more tutors like you.

  • @m.donnediego587
    @m.donnediego587 5 лет назад +27

    I like how you subtitle what you speak, it's helpful for me.

  • @wssz112
    @wssz112 8 лет назад +65

    dam now i want to go to MIT

  • @ianmoseley9910
    @ianmoseley9910 7 лет назад +3

    makes me realise how much I have forgotten in the 40+ years since uni. Expressing the integral in that graphic form was not something we were taught and it really helps me to grasp the whole concept; would have grokked so much more if we had been!

  • @rthelionheart
    @rthelionheart 3 года назад +7

    In my school they simply say: evaluate the following integral just as given in part (a) above. It is entirely ALL UP TO YOU how you approach it. Of course one method makes life a whole lot easier than say compared to another one. It is expected that you know which method to use accordingly.

    • @beoptimistic5853
      @beoptimistic5853 3 года назад

      ruclips.net/video/vFDMaHQ4kW8/видео.html ...💐

  • @VocalBeast
    @VocalBeast Год назад +1

    Man thank you so much, literally the first time i actual properly understood polar coordinate transformations despite trying to wrap my head around it for like 3 months. Appreciate you!!!

  • @benn7139
    @benn7139 7 лет назад +168

    his chalk is so big

  • @AldenRyno
    @AldenRyno 13 лет назад +3

    @marcuswauson
    The integrand is [cos(theta)-(1/2)*cos(theta)] but when integrated, it becomes [sin(theta)-(1/2)*sin(theta)] from zero to pi/4. When evaluated equals sqrt(2)/2 - sqrt2)/4 = sqrt(2)/4

  • @muhammadzeeshankhan7251
    @muhammadzeeshankhan7251 8 лет назад +9

    Beautifully explained,
    I liked your teaching.

  • @morgard211
    @morgard211 3 года назад +4

    These are actually very instructive excercises.

    • @beoptimistic5853
      @beoptimistic5853 3 года назад

      ruclips.net/video/vFDMaHQ4kW8/видео.html ...💐

  • @ashtongaeta2581
    @ashtongaeta2581 4 года назад +8

    Thank you I have to study at home because of the corona virus. This came in so clutch!!

    • @mitocw
      @mitocw  4 года назад +8

      Happy to help!

  • @DeprecatedAPI
    @DeprecatedAPI 6 лет назад +12

    This was really awesome, thanks! Helped me in my end sems!!

  • @oliverbrace4505
    @oliverbrace4505 11 лет назад +6

    dude, you`re awesome, i was able to do my homework thanks to you

  • @anikethsridhargund6671
    @anikethsridhargund6671 2 года назад +2

    Thanks a lot, I was struggling to solve a problem related usage of polar coordinates in my assignment, but now i solved it in just 5min. thanks once again!

  • @suruchisolanki3603
    @suruchisolanki3603 3 года назад +2

    Thanks a lot MIT. I've finally understood the concept!!!

  • @shreyassahu6930
    @shreyassahu6930 5 лет назад +1

    This is one of the best explanations for this topic.

  • @meerismailali6082
    @meerismailali6082 2 года назад +2

    You don’t just cancel the r in 10:45. You solve for quadratic

  • @rkumaresh
    @rkumaresh 6 лет назад +1

    What a simple explanation which everyone could understand

  • @alexandresauve7560
    @alexandresauve7560 11 лет назад

    yes; and this is how we deal with it. find the range of angles for which r is negative, split these segments away from your first integral and put a negative sign in front in order to find the area. example r=cos(theta) from 0 to 2pi gives negative r from pi/2 to 3pi/2. the area enclose by r=cos(theta) = integral of rcos(thta) from 3pi/2 to pi/2 plus (-) integral cos(thta) from pi/2 to 3pi/2. integrating the original function from o to 2pi gives 0 if you ignore the fact r goes negative.

  • @sairajbhosale3984
    @sairajbhosale3984 8 месяцев назад

    This was so helpful to solve questions.
    My professor just solve 3-4 easy questions and left us with such questions
    Thanks a lot sir ..

  • @Jameel263
    @Jameel263 13 лет назад

    @PeaceUdo
    Question a and b) The upper bound for y is y=x.
    The line y = x is always at a 45 degree (pi/4) angle with the x axis.
    If you dont get why, then for example lets say y = x = n (as y=x)
    then
    tan θ = n/n
    tan θ = 1
    therefore θ=45 degree (pi/4)

  • @MCSPT117
    @MCSPT117 12 лет назад +2

    Because he has (1/r^3) r drdΘ, if you combine that r with (1/r^3) it would have a negtive power if you put it over one, (r has a power of one), therefore it becomes (1/r^(3-1)... and becomes 1/r^2. Hope this helps.

    • @23StudiosSports
      @23StudiosSports 2 года назад

      That helps sp much you have no idea, you are a legend.

  • @st.johntsuno-wayne2489
    @st.johntsuno-wayne2489 2 года назад

    only in Calc BC trying to solve an argument with a friend when i found this… video made the concepts simple to understand and was extremely informative!

  • @Paulovrish7334
    @Paulovrish7334 5 лет назад +1

    Finally, it makes perfect sense

  • @LICKSandWINKS
    @LICKSandWINKS 11 лет назад

    I have an advanced calc exam on Tuesday AND YOU ARE A LIFE SAVER ILY

  • @edwinsebastianperezrodrigu8659
    @edwinsebastianperezrodrigu8659 3 года назад

    First time i watch a MIT class and i understand everything

  • @vrendus522
    @vrendus522 11 лет назад +1

    To the instructor, thank you.

  • @NickZachPattyWack
    @NickZachPattyWack 12 лет назад

    Not only do the brilliant students of MIT get excellent student teachers, but I can't understand majority of what my teacher says through his thick accent...and no my school is not well known even in the city that it resides in. Thanks David and MIT!

  • @DoggoWillink
    @DoggoWillink 12 лет назад +2

    yeah, he didn't realize it was 1/r^3 when he re-wrote it

  • @anuraagkaravadi1640
    @anuraagkaravadi1640 9 лет назад +1

    limits explanation was awesome --which obviously is the heart and soul of the prob...lol in C in forgot to put square root.........

  • @HimanshuShekhar13s
    @HimanshuShekhar13s 5 лет назад +1

    Mindblowing explanation! Thanks!

  • @quratulainfatima109
    @quratulainfatima109 7 лет назад +1

    Explained beautifully........

  • @GradientSoln-En
    @GradientSoln-En 7 месяцев назад

    Thank you, just thank you.

  • @gnauhandy
    @gnauhandy 13 лет назад +1

    @marcuswauson you're evaluating 1/2 sin theta from 0-Pi/4.

  • @jbonn5
    @jbonn5 12 лет назад

    @maplestorypl He wrote it correctly. dA becomes r dr dtheta so the integral does become 1 / r^2.

  • @bhaktimd9299
    @bhaktimd9299 4 года назад +2

    What can be his age in 2011🤔

  • @joeferreira-qr7iq
    @joeferreira-qr7iq Год назад

    the one thing i think that is missing when converting is that the F function at the end is not a function of x, y but r, theta
    you cant just plug in f given a function of x,y when in polar form

  • @usamafarooqi7292
    @usamafarooqi7292 2 года назад

    Brillient.... Absolutly great

  • @vychuck
    @vychuck 9 лет назад +1

    Well done young man, really nailed it

  • @MayankSharma-qd9ny
    @MayankSharma-qd9ny 3 года назад

    Finally understood the concept

  • @platinumk
    @platinumk 12 лет назад +1

    nice and concise. thank you david

  • @joshie92
    @joshie92 12 лет назад

    the -(1/r) can be changed to (1/r) if you swap the limits of integration, which is what he did here.

  • @Dhavalc2011
    @Dhavalc2011 11 лет назад +2

    Best ! Like IT ! Simply ,straight forward and lucid :D

  • @WolfBoyBenRawr
    @WolfBoyBenRawr 12 лет назад +3

    Thank you David!! This was extremely helpful !

  • @AbhayArsekar
    @AbhayArsekar 5 лет назад +1

    thank you so much David ❤️

  • @gabrieltmapondera9697
    @gabrieltmapondera9697 7 лет назад +15

    how is dxdy equal to rdrdθ

    • @adellewilliams3266
      @adellewilliams3266 6 лет назад +9

      to prove this , we use jacobian method. And we all know that from cartessian to polar coordinate , x=cos@ and y=sin@ then dxdy change to drd@ and apply this to jacobian matrix (dx/dr dx/d@ , dy/dr dy/d@) then we get rdrd@

    • @molkgfmf5699
      @molkgfmf5699 6 лет назад +1

      or... dA = (dr) (ds)? = (dr) d(rθ) = r (dr) (dθ) = (dx) (dy)

    • @AkashYadav-mr4hg
      @AkashYadav-mr4hg 6 лет назад +1

      use Jacobian

    • @JamilKhan-hk1wl
      @JamilKhan-hk1wl 5 лет назад +1

      There is a simpler method to understand that, dxdy is actually a very small square. So its equal to rdrdtheta

    • @АзХашми
      @АзХашми 5 лет назад +1

      Look up the Jacobian for polar coordinates

  • @vinitamaharaj5738
    @vinitamaharaj5738 7 лет назад +3

    Really helpful, thank you!

  • @marcuswauson
    @marcuswauson 13 лет назад

    cos of 0 is not 0, cos of 0 is 1, your answer to a should be (squareroot of 2) minus 2 all over 4. As cos of 0 is one, and one minus a half should give you squareroot of 2 over four minus a half.

  • @rain74925
    @rain74925 13 лет назад +6

    Thanks! I think you articulate it really well. That's awsome! Wish you're my prof.
    Not to be disrespectful, but are you an undergraduate? You look like a peer.

    • @erikumble
      @erikumble 3 года назад

      In the introduction video of the TAs, David was a graduate student. Of course, that was 10 years ago.

  • @QuantumDisciple7
    @QuantumDisciple7 12 лет назад +1

    Very nicely demonstrated. I appreciate it!

  • @AhmadKhan-hb3je
    @AhmadKhan-hb3je 6 лет назад +2

    Nicely explained. But getting out of the board feels very awkward.

  • @HomoSapiensMember
    @HomoSapiensMember 5 лет назад +2

    this helped me, thanks so much!

  • @hargeysasomaliland4374
    @hargeysasomaliland4374 12 лет назад +1

    Best lecture . Thanks for your kindly helping

  • @VikasSingh-cv2fu
    @VikasSingh-cv2fu 7 лет назад +1

    Good job bro. You really explain well.

  • @pedroff_1
    @pedroff_1 6 лет назад +2

    I think somewhere in the middle of a.), you accidentally replaced 1/r^3 with 1/r^2 ...

    • @sumeepriya5454
      @sumeepriya5454 4 года назад

      Watch it again its correct

    • @ChucksSEADnDEAD
      @ChucksSEADnDEAD 4 года назад +1

      It was subtle but you have to watch out for the 1/r^3 r dr dtheta. The r in the nominator position crosses out one of the rs in the denominator leaving 1/r^2 dr dtheta.

  • @syoushiro1837
    @syoushiro1837 6 лет назад

    at 10:33, you should not cancel the variable r because you miss the answer r = 0.

  • @sumitgupta6905
    @sumitgupta6905 6 лет назад

    Best online classroom

  • @riya6549
    @riya6549 3 года назад +1

    is this the same as multiple integrals? im just starting out and im very confused...

    • @beoptimistic5853
      @beoptimistic5853 3 года назад

      ruclips.net/video/vFDMaHQ4kW8/видео.html ...💐

  • @русскийпартизан-ь6п

    I loved it! Thanks so much!!!

  • @femiairboy94
    @femiairboy94 7 лет назад +1

    So how do you solve this if e^-x^2 is multiplied by Cos(bx) with respect to x

  • @ERLAANKABABU23BME706
    @ERLAANKABABU23BME706 8 месяцев назад

    He is a genius...

  • @nelsonlopez9587
    @nelsonlopez9587 5 лет назад +3

    MIT depends heavily on the chalk industry

  • @moasfco11
    @moasfco11 13 лет назад +1

    thanks very helpful examples

  • @anjaneyasharma322
    @anjaneyasharma322 2 года назад

    Students note double or triple integration does not give correct results.
    Follow simple integration.
    The question is why count something twice as it happens in double triple integration.

  • @the_eternal_student
    @the_eternal_student 15 дней назад

    How did he know 1/(x^2+y^2)^1/2 was r?
    How did he know x^2+y^2=r^2?

  • @BlazeCyndaquil
    @BlazeCyndaquil 11 лет назад +1

    On the first one, shouldn't the result have been sqrt(2)/4 - 1/2?
    Wouldn't you have to subtract off the (1 - 1/2) from the lower bound of theta?

  • @ananyarathore4678
    @ananyarathore4678 2 года назад

    loveddd ittt!!! thankyouuu so much david sir

  • @pudingstorm1
    @pudingstorm1 8 лет назад +1

    Than you man, great video, simple and nice explanation :D

  • @prakharuttarpradesh3517
    @prakharuttarpradesh3517 Год назад +2

    Thank you professor You cleared all my doubts
    🙏🙏 Dhanyawad and Namaskar

  • @rationalLeft
    @rationalLeft 10 лет назад +4

    I think there is a little mistake in the end of letter a). The result should be sqrt (2)/ 4 - 1/2, instead of sqrt (2)/4. Thank you for the excelent explanation.

    • @waltvanamstel6807
      @waltvanamstel6807 10 лет назад +7

      you have to integrate the 1/2*cos(θ), which gives you 1/2*sin(θ) integrating from 0 to pi/4. So sqrt(2)/4 is indeed correct.

    • @samuelminea5520
      @samuelminea5520 7 лет назад

      rationalLeft He had to integrate cos(a)-1/2*cos(a)=1/2*cos(a) which has the antiderivative equal to 1/2*sin(a) from 0 to pi/4 = sqrt(2)/(4)

  • @mustaphabouchaqour7770
    @mustaphabouchaqour7770 5 лет назад

    for the last problem, it would be easy to switch the coordinate instead of x give y and so on.

  • @freakingik2781
    @freakingik2781 2 года назад

    Welcome back

  • @MarikoIchigo
    @MarikoIchigo 12 лет назад +1

    shouldn´t teta go from -π/2 to π/2?

  • @thecivilizedguy8425
    @thecivilizedguy8425 4 года назад

    Very clear teaching!

  • @darprahimi9692
    @darprahimi9692 7 лет назад +1

    Final answer for c) ??

  • @deepakkumarchandel2444
    @deepakkumarchandel2444 3 года назад

    Excellent no one able to tell how limits of polar are going on.🙏

  • @johnpwnsyou
    @johnpwnsyou 11 лет назад

    No. There are several reasons why. First, r is a length, which logically can't be negative. Second, If you look at the formula, r=(x^2+y^2)^(1/2), you'll realize that there's no way for it to be negative since it's components are all squared.

    • @valkarez1137
      @valkarez1137 5 лет назад

      youre stupid

    • @sreekarg9553
      @sreekarg9553 5 лет назад

      You’re replying to another comment aren’t you?

  • @ricardinhovorkes4876
    @ricardinhovorkes4876 6 лет назад

    Thank you thank you thank you I appreciate it a lot. Nice instructor very helpful

  • @celsiusfahrenheit1176
    @celsiusfahrenheit1176 2 года назад

    ARCTAN(y/x) when y==x is π/4 for the rest of us

  • @kamaraju1551
    @kamaraju1551 12 лет назад +1

    SUPER!

  • @zeldadu09
    @zeldadu09 11 лет назад +1

    Great Video!!! Greetings from GT!!!

  • @Dineshkumar-xv4xz
    @Dineshkumar-xv4xz 5 лет назад

    Awesome thank you so much mit

  • @dant9944
    @dant9944 6 лет назад

    why in example c teta goes only to Pi/2 = 90 and not to pi =180

  • @noahmckeever9059
    @noahmckeever9059 9 лет назад +1

    In part a) why do you go from 1/r^3 to 1/r^2?

    • @AlexVX_
      @AlexVX_ 9 лет назад

      he multiplied 1/r^3 by the r from "r dr d theta", leaving 1/r^2

  • @Lagos3sgte
    @Lagos3sgte 12 лет назад +1

    Great video! Really helpful.

  • @anannyauberoi7984
    @anannyauberoi7984 8 лет назад +1

    Thanks a ton!

  • @FirstGradeCalculus
    @FirstGradeCalculus 12 лет назад

    Great videos, David. Thanks kindly!

  • @matthewskatuta1305
    @matthewskatuta1305 3 года назад

    Excellent work

  • @krishnaghorai8146
    @krishnaghorai8146 5 лет назад

    Good base ,so love you sir!

  • @animeshpathak3921
    @animeshpathak3921 5 лет назад

    how could you just assume the lower limit oangle of the parabola to be 0

  • @aditkalyani7966
    @aditkalyani7966 6 лет назад

    Very well explained!

  • @HereToday32
    @HereToday32 10 лет назад +2

    how do you know the maximum line is pi/4?

    • @jeffreychang2293
      @jeffreychang2293 10 лет назад +5

      Look at a unit circle. You see that y = x is half of quadrant one and quadrant one is from 0 to pi/2. and half of that is pi/4 so you get theta = pi/4 as your max theta value.

  • @hasanabs
    @hasanabs Год назад

    Does the last example diverge? Because the the internal integration integral[r=0,r=2sin(theta)] r^-2 dr = -1/r | [r=0,r=2sin(theta)] = -(1/2sin(theta)-1/0) = diverge

  • @_tasneem7378
    @_tasneem7378 2 года назад

    That was really helpful. Super clear!

  • @l1mmg0t
    @l1mmg0t 3 года назад

    I am an old man now. seems I can understand this better than my young age. feel like to go back to school.

  • @grovestreet9165
    @grovestreet9165 6 лет назад +3

    Perfect

  • @beercity123
    @beercity123 13 лет назад

    Worth the watch! Very helpful!

  • @sajathsalim261
    @sajathsalim261 5 лет назад

    Oh my gosh.. Our IIT is world's best in terms of proffessor knowledge

    • @MohitSharma-wt9ex
      @MohitSharma-wt9ex 5 лет назад +2

      bro iit nahi VO MIT ka he .
      aur MIT no 1 he world me

  • @nitesjung1274
    @nitesjung1274 3 года назад

    Thank you so much kind sir.

    • @beoptimistic5853
      @beoptimistic5853 3 года назад

      ruclips.net/video/vFDMaHQ4kW8/видео.html ...💐

  • @sumballaboi
    @sumballaboi 12 лет назад +1

    great video mate, really helpful!!